0 Daumen
139 Aufrufe

Aufgabe:

Die Produktionsfunktion eines Unternehmens laute
\( F\left(x_{1}, x_{2}\right)=10 x_{1}^{2}+62 x_{1} x_{2}+10 x_{2}^{2}, \)
wobei \( x_{1} \) und \( x_{2} \) die eingesetzten Mengen der beiden Produktionsfaktoren \( \boldsymbol{A} \) und \( \boldsymbol{B} \) bezeichnen. Die Kosten der Produktionsfaktoren betragen pro Mengeneinheit 63 bzw. 77 Geldeinheiten. Vom Endprodukt sollen 7424 Mengeneinheiten gefertigt werden. Für die Produktionskosten in Abhängigkeit von den eingesetzten Mengen der beiden

Produktionsfaktoren \( A \) und \( B \) existiert unter dieser Nebenbedingung im ersten Quadranten genau eine lokale Extremstelle. Ermitteln Sie die folgenden Größen:
a. Bei welcher Menge von \( x_{1} \) werden bei einem Output von \( 7424 \mathrm{ME} \) die Kosten minimal?
b. Bei welcher Menge von \( x_{2} \) werden bei einem Output von 7424 ME die Kosten minimal?
c. Welchen Wert hat der Lagrange-Multiplikator \( \lambda \) im Kostenminimum?
d. Wie lautet das kostenminimale
Faktoreinsatzverhältnis der beiden
Produktionsfaktoren \( x_{1} \) und \( x_{2} ? \)


Problem/Ansatz


Kann diese Aufgabe nicht lösen, bitte um Hilfe wie ich hier zu den Ergebnissen komme

Avatar von

2 Antworten

+1 Daumen
 
Beste Antwort

Aloha :)

Zur Optimierung des Kostenfunktion \(c(x;y)\) unter einer konstanten Nebenbedinung \(f(x;y)\)$$c(x;y)=63x+77y\quad;\quad f(x;y)=10x^2+62xy+10y^2\stackrel!=7424=\text{const}$$kannst du das Lagrange-Kriterium nutzen. Demnach ist in einem Extremum der Gradient der zu optimierenden Funktion eine Linearkombination der Gradienten aller konstanten Nebenbedingungen. Da es hier nur eine Nebenbedingung gibt, heißt das:$$\operatorname{grad}c(x;y)=\lambda\cdot\operatorname{grad}f(x;y)\implies\binom{63}{77}=\lambda\binom{20x+62y}{62x+20y}$$

Um den Lagrange-Multiplikator \(\lambda\) loszuwerden, divdieren wir die Gleichung der ersten Koordinate durch diejenige der zweiten Koordinate:$$\frac{63}{77}=\frac{\lambda(20x+62y)}{\lambda(62x+20y)}\implies\frac{9}{11}=\frac{10x+31y}{31x+10y}\implies9(31x+10y)=11(10x+31y)$$$$\implies279x+90y=110x+341y\implies169x=251y\implies \underline{\underline{y=\frac{169}{251}\cdot x}}$$

Damit ist das Optimierungsproblem gelöst und wir können die Fragen beantworten.

a. Bei welcher Menge von \(x_{1}\) werden bei einem Output von 7424 ME die Kosten minimal?

Dazu setzen wir die Lagrange-Bedingung in die konstante Nebenbedingung ein:$$7424=10x^2+62x\cdot\frac{169}{251}\,x+10\left(\frac{169}{251}\,x\right)^2=\frac{3\,545\,598}{63\,001}\,x^2\implies$$$$x=\sqrt{\frac{7424\cdot63\,001}{3\,545\,598}}\implies\boxed{x\approx11,48544843}$$Die negative Lösung (Wurzelziehen!) fällt weg, da es keine negativen Produktionsmengen gibt.

b. Bei welcher Menge von \( x_{2} \) werden bei einem Output von 7424 ME die Kosten minimal?

Dazu setzen wir das in (a) bestimmte \(x\) in die Lagrange-Bedingung ein:$$y=\frac{169}{251}\cdot11,48544843\implies \boxed{y\approx7,73323022}$$

c. Welchen Wert hat der Lagrange-Multiplikator \( \lambda \) im Kostenminimum?

Da wir nun \(x\) und \(y\) im Optimum kennen, wählen wir die erste Koordinatengleichung der Gradienten-Bedingung, um \(\lambda\) zu bestimmen:$$63=\lambda(20x+62y)\implies\lambda=\frac{63}{20x+62y}\implies \boxed{\lambda\approx0,08883634}$$

d. Wie lautet das kostenminimale Faktoreinsatzverhältnis der beiden Produktionsfaktoren \( x_{1} \) und \( x_{2} ? \)

Das folgt direkt aus der Lagrange-Bedingung:$$\frac yx=\frac{169}{251}\quad\text{oder}\quad\frac xy=\frac{251}{169}$$Ich weiß nicht genau, welches dieser beiden Verhältnisse gesucht ist, deswegen habe ich keins als Ergebnis eingerahmt. Da musst du mal in deine Vorlesung gucken, wie ihr das vereinbart habt.

Avatar von 148 k 🚀
0 Daumen

minimiere 63x1 + 77x2 unter der Nebenbedingung F(x1, x2) = 7424

Wie man das tun kann, steht in Frage c).

Avatar von 43 k

Ein anderes Problem?

Stell deine Frage

Willkommen bei der Mathelounge! Stell deine Frage einfach und kostenlos

x
Made by a lovely community